Question

· Question 4 Assuming a down-sloping demand curve and positive marginal cost, total revenue (TR) maximization...

· Question 4

Assuming a down-sloping demand curve and positive marginal cost, total revenue (TR) maximization requires:

  1. TR = 0
  2. ATC = 0
  3. MR = 0
  4. Aπ = 0

· Question 5

Assuming a down-sloping demand curve and positive marginal cost, the output at the maximum total profit (Tπ):

  1. Is less than that at the maximum total revenue
  2. Is greater than that at the maximum total revenue
  3. Can be greater or less than that at the maximum total revenue
  4. Occurs at MR=0

· Question 6

In the long-run, a firm:

  1. Can minimize loss by shutting down its plant
  2. Making continuing losses should leave the industry
  3. May produce even it TR is less than TC at all output levels
  4. May not have enough time to leave and enter the industry

Homework Answers

Answer #1

Question 4. Option C is correct. Total revenue is maximum when the marginal revenue is zero. This is because marginal revenue is the derivative of total revenue. Therefore when finding the maxima, marginal revenue will be zero

Question 5. Option A is correct. profit maximizing level of output is always less than the revenue maximizing level of output. This is because when revenue is maximum, marginal revenue zero and marginal cost is positive. This indicates that output level that maximizes profit has already reached

Question 6. Option B is correct . This is because when there are consistent losses firm can leave the industry and save the fixed cost.

Know the answer?
Your Answer:

Post as a guest

Your Name:

What's your source?

Earn Coins

Coins can be redeemed for fabulous gifts.

Not the answer you're looking for?
Ask your own homework help question
Similar Questions
economic of health and medical care In a MS Word document, define total revenue (TR), marginal...
economic of health and medical care In a MS Word document, define total revenue (TR), marginal revenue (MR), and the profit-maximizing rule for a single investor-owned firm. Then calculate MR, MC, and ATC for Table 3.1. Next, give the profit-maximizing level of output (Q). Now, assume the firm is a tax-exempt agency. One possibility is that tax-exempt agencies maximize output. Define the output-maximization rule and then give the output-maximizing level of output (Q) given Table 3.1. What happens to the...
Barbara is a producer in a monopoly industry. Her demand curve, total revenue curve, marginal revenue...
Barbara is a producer in a monopoly industry. Her demand curve, total revenue curve, marginal revenue curve, and total cost curve are given as follows Q = 160 – 4P TR – 40Q – 0.25Q² MR = 40 – 0.5Q TC = 4Q MC = 4 Answer the following three questions: (You don't need to show your work, just type your answer as a number) a) What is the optimum output for Barbara? b) What is the market price? c)...
Q                  TR              MR             
Q                  TR              MR                  TC                             MC                             ATC 0                     0                -                       100                            -                                   - 1                   200            200                    200                         100                               200 2                   400              200                   350                          150                              175 3                   600              200                  550                          200                               183.3 4                   800              200                   800                          250                               200 5                   1000            200                   1100                        300                               220 Quantity of Visits (Q) Total Revenue (TR) Marginal Revenue (MR) Total Costs (TC) Marginal Cost (MC) Average Total Cost (ATC) In a MS Word document, define total revenue (TR), marginal revenue (MR), and the profit-maximizing rule for...
A monopolist faces the following demand curve, marginal revenue curve, total cost curve and marginal cost...
A monopolist faces the following demand curve, marginal revenue curve, total cost curve and marginal cost curve for its product: Q = 200 - 2P MR = 100 - Q    TC = 5Q MC = 5    a. What is the profit maximizing level of output? b. What is the profit maximizing price? c. How much profit does the monopolist earn?
For a linear demand curve, if MR = 0: Select one: a. TR is at its...
For a linear demand curve, if MR = 0: Select one: a. TR is at its minimum. b. and TR is at its maximum, the demand curve must be downward sloping. c. all of the above are true. d. the price elasticity of demand is equal to or greater than -1.0.
Show the mathematical argument that for a monopolist who faces a downward-sloping demand curve, marginal revenue...
Show the mathematical argument that for a monopolist who faces a downward-sloping demand curve, marginal revenue is less than price whenever quantity sold is positive.
Assume the demand schedule below and TC function below to find maximize profit, Total Revenue (TR),...
Assume the demand schedule below and TC function below to find maximize profit, Total Revenue (TR), Total Cost(TC), and max profit. Using the MR=MR (Marginal Revenue) approach or Excel (Solver). P = 10 - .5Q TC = 10 + Q - .4Q2
If average total cost (ATC) curve is always downward sloping, then A. marginal cost will cross...
If average total cost (ATC) curve is always downward sloping, then A. marginal cost will cross ATC at its minimum B. average variable cost will cross ATC at its minimum C. the firm has economies of scale D. average fixed cost will cross ATC at its minimum Given a U-shaped Average Total Cost (ATC) curve A. Average Fixed costs will always be above ATC B. Marginal cost will always be below ATC C. Average variable costs will always be below...
Q) Perfect Competition Demand: P=$4 Marginal revenue: MR = $4 Average total cost: ATC = 2/Q...
Q) Perfect Competition Demand: P=$4 Marginal revenue: MR = $4 Average total cost: ATC = 2/Q + Q Marginal cost :MC = 2Q Draw a graph showing MC. MR, demand, and ATC. Illustrate this firm's revenue, cost, and profit in your graph. Then, Explain why the demand is perfectly elastic in a perfectly competitive marker.
4.Which statement isincorrect? a.A pure monopolist’s demand curve is the market demand curve. b.A monopoly produces...
4.Which statement isincorrect? a.A pure monopolist’s demand curve is the market demand curve. b.A monopoly produces a product for which there are no close substitutes. c.Marginal revenue is less than price for a monopolist that cannot price discriminate. d.A monopolist’s market position ensures positive economic profits. 5.For a firm with monopoly power that cannot engage in price discrimination: a.the marginal revenue curve lies below the demand curve because any reduction in price applies only to the last unit sold. b.the...